Historian

This topic has expert replies
User avatar
Master | Next Rank: 500 Posts
Posts: 166
Joined: Fri Apr 03, 2009 11:16 pm
Location: India

Historian

by Mayur Sand » Thu Jul 23, 2009 6:40 pm
Historian: In the Drindian Empire, censuses were conducted annually to determine the population of each village. Village census records for the last half of the 1600’s are remarkably complete. This very completeness makes one point stand out; in five different years, villages overwhelmingly reported significant population declines. Tellingly, each of those five years immediately followed an increase in a certain Drindian tax. This tax, which was assessed on villages, was computed by the central government using the annual census figures. Obviously, whenever the tax went up, villages had an especially powerful economic incentive to minimize the number of people they recorded; and concealing the size of a village’s population from government census takers would have been easy. Therefore, it is reasonable to think that the reported declines did not happen.

In the historian’s argument, the two portions in boldface play which of the following roles?

A. The first supplies a context for the historian’s argument; the second acknowledges a consideration that has been used to argue against the position the historian seeks to establish.
B. The first presents evidence to support the position that the historian seeks to establish; the second acknowledges a consideration that has been used to argue against that position.
C. The first provides a context for certain evidence that supports the position that the historian seeks to establish; the second is that position.
D. The first is a position for which the historian argues; the second is an assumption that serves as the basis of that argument.
E. The first is an assumption that the historian explicitly makes in arguing for a certain position; the second acknowledges a consideration that calls that assumption into question.

I went for E but OA is C Please explain

Legendary Member
Posts: 882
Joined: Fri Feb 20, 2009 2:57 pm
Thanked: 15 times
Followed by:1 members
GMAT Score:690

by crackgmat007 » Thu Jul 23, 2009 6:58 pm
C. The first provides a context for certain evidence that supports the position that the historian seeks to establish; --> The first is introduced to set a contenxt that is used to explain why census was complete and the motive behind understating.

the second is that position. --> Conclusion of the historian as indicated by 'Therefore'

E. The first is an assumption that the historian explicitly makes in arguing for a certain position; --> Although it is used to set the context, it may be treated as an assumption.

the second acknowledges a consideration that calls that assumption into question. --> Assumption is not called into question. Conclusioon is that reported declines did not happen. If the assumption were reported declines did happen, then this would have been correct. Assumption here is about completeness but not about decline in census.

HTH

Legendary Member
Posts: 503
Joined: Sun Aug 09, 2009 9:53 pm
Thanked: 31 times
Followed by:2 members

by mmslf75 » Tue Jan 12, 2010 10:55 am
is this argument proper ?

GMAT Instructor
Posts: 1302
Joined: Mon Oct 19, 2009 2:13 pm
Location: Toronto
Thanked: 539 times
Followed by:164 members
GMAT Score:800

by Testluv » Tue Jan 12, 2010 5:39 pm
We can answer this question just by noticing that the second bold statement is a conclusion (because it follows "therefore").

Only choice C correctly states that the second bold statement is a conclusion ("a position").

Therefore, choice C must be correct.

As an aside, a statement can NEVER be an assumption because, by definition, an assumption is unstated.

@mmslf: why do you think the argument might not be proper?
Kaplan Teacher in Toronto

Legendary Member
Posts: 503
Joined: Sun Aug 09, 2009 9:53 pm
Thanked: 31 times
Followed by:2 members

by mmslf75 » Tue Jan 12, 2010 8:11 pm
Testluv wrote:We can answer this question just by noticing that the second bold statement is a conclusion (because it follows "therefore").

Only choice C correctly states that the second bold statement is a conclusion ("a position").

Therefore, choice C must be correct.

As an aside, a statement can NEVER be an assumption because, by definition, an assumption is unstated.

@mmslf: why do you think the argument might not be proper?
Hi Testluv,

What I meant was :
On one hand argument mentions that, village census records for the last half of the 1600's are remarkably complete. This very completeness makes one point stand out;
On the other hand villages had an especially powerful economic incentive to minimize the number of people they recorded;

IF village adopted such an incentive then the census records for last half of 1600's should NOT have been remarkbly COMPLETE

I guess I am missing something here ?
Can you please point out :-)

GMAT Instructor
Posts: 1302
Joined: Mon Oct 19, 2009 2:13 pm
Location: Toronto
Thanked: 539 times
Followed by:164 members
GMAT Score:800

by Testluv » Tue Jan 12, 2010 11:36 pm
Oh, yes, I see how that can be confusing. I interepreted "remarkably complete" to mean "remarkably thorough" given that they are records from the 1600s. Here, "remarkably complete" doesn't mean that they counted every person.
Kaplan Teacher in Toronto

Senior | Next Rank: 100 Posts
Posts: 31
Joined: Fri Jun 12, 2009 8:43 am

by gmattering » Thu Jan 14, 2010 6:30 am
Mayur Sand wrote:Historian: In the Drindian Empire, censuses were conducted annually to determine the population of each village. Village census records for the last half of the 1600�s are remarkably complete. This very completeness makes one point stand out; in five different years, villages overwhelmingly reported significant population declines. Tellingly, each of those five years immediately followed an increase in a certain Drindian tax. This tax, which was assessed on villages, was computed by the central government using the annual census figures. Obviously, whenever the tax went up, villages had an especially powerful economic incentive to minimize the number of people they recorded; and concealing the size of a village�s population from government census takers would have been easy. Therefore, it is reasonable to think that the reported declines did not happen.

In the historian�s argument, the two portions in boldface play which of the following roles?

A. The first supplies a context for the historian�s argument; the second acknowledges a consideration that has been used to argue against the position the historian seeks to establish.
B. The first presents evidence to support the position that the historian seeks to establish; the second acknowledges a consideration that has been used to argue against that position.
C. The first provides a context for certain evidence that supports the position that the historian seeks to establish; the second is that position.
D. The first is a position for which the historian argues; the second is an assumption that serves as the basis of that argument.
E. The first is an assumption that the historian explicitly makes in arguing for a certain position; the second acknowledges a consideration that calls that assumption into question.

I went for E but OA is C Please explain

the passage seems to be flowing smoothly in agreement with no "contrasting" elements so i took out A, B, E which contain the word "against".

I narrowed to C because D mentioned "argues" - there is no element of arguement in the passage

GMAT Instructor
Posts: 1302
Joined: Mon Oct 19, 2009 2:13 pm
Location: Toronto
Thanked: 539 times
Followed by:164 members
GMAT Score:800

by Testluv » Thu Jan 14, 2010 10:42 am
I narrowed to C because D mentioned "argues" - there is no element of arguement in the passage
As I wrote in my post above, we don't even need to read the answer choices completely in order to select the correct answer here. All we need to see is that the second bold statement is definitely a conclusion. When I saw this, I went to the answer choices, reading ONLY the second clauses of each choice, eliminating those that didn't say "conclusion"...I was a bit surprised that I was able to eliminate all four wrong answers this way only choice C states the second bold statement is a conclusion.
Kaplan Teacher in Toronto

Senior | Next Rank: 100 Posts
Posts: 38
Joined: Wed Jul 22, 2009 8:49 am
Thanked: 4 times

by rahulmehra13 » Sun Jun 06, 2010 12:26 am
Testluv wrote:We can answer this question just by noticing that the second bold statement is a conclusion (because it follows "therefore").

Only choice C correctly states that the second bold statement is a conclusion ("a position").

Therefore, choice C must be correct.

As an aside, a statement can NEVER be an assumption because, by definition, an assumption is unstated.

@mmslf: why do you think the argument might not be proper?
@Testluv, check this Q:

Although the earliest surviving Greek inscriptions written in an alphabet date from the eighth century B.C., a
strong case can be made that the Greeks actually adopted alphabetic writing at least two centuries
earlier. Significantly, the text of these earliest surviving Greek inscriptions sometimes runs from right to left and
sometimes from left to right. Now, the Greeks learned alphabetic writing from the Phoenicians, and in the
process they would surely have adopted whatever convention the Phoenicians were then using with
respect to the direction of writing. Originally, Phoenician writing ran in either direction, but by the eighth
century B.C. it had been consistently written from right to left for about two centuries. In the argument given,
the two portions in boldface play which of the following roles?

A. The first is the position that the argument seeks to establish; the second reports a discovery that has been used
to support a position that the argument opposes.
B. The first is the position that the argument seeks to establish; the second presents an assumption on which the
argument relies.
C. The first presents evidence that is used in support of the position that the argument seeks to establish; the
second presents an assumption on which the argument relies.
D. The first is an objection raised against a position that the argument opposes; the second is the position that the
argument seeks to establish.
E. The first is an objection raised against a position that the argument opposes; the second is evidence that has
been used to support that position.

OA is B, you said 'a statement can NEVER be an assumption' but in this case it is. Can you please shed some light.

Master | Next Rank: 500 Posts
Posts: 134
Joined: Mon Mar 01, 2010 7:08 am
Thanked: 3 times

by bupbebeo » Sun Jun 06, 2010 12:37 am
Mayur Sand wrote:Historian: In the Drindian Empire, censuses were conducted annually to determine the population of each village. Village census records for the last half of the 1600�s are remarkably complete. This very completeness makes one point stand out; in five different years, villages overwhelmingly reported significant population declines. Tellingly, each of those five years immediately followed an increase in a certain Drindian tax. This tax, which was assessed on villages, was computed by the central government using the annual census figures. Obviously, whenever the tax went up, villages had an especially powerful economic incentive to minimize the number of people they recorded; and concealing the size of a village�s population from government census takers would have been easy. Therefore, it is reasonable to think that the reported declines did not happen.

In the historian�s argument, the two portions in boldface play which of the following roles?

A. The first supplies a context for the historian�s argument; the second acknowledges a consideration that has been used to argue against the position the historian seeks to establish.
B. The first presents evidence to support the position that the historian seeks to establish; the second acknowledges a consideration that has been used to argue against that position.
C. The first provides a context for certain evidence that supports the position that the historian seeks to establish; the second is that position.
D. The first is a position for which the historian argues; the second is an assumption that serves as the basis of that argument.
E. The first is an assumption that the historian explicitly makes in arguing for a certain position; the second acknowledges a consideration that calls that assumption into question.

I went for E but OA is C Please explain

E wrong because an assumption is unstated premises, so it can appear explicitly on the argument.

GMAT Instructor
Posts: 1302
Joined: Mon Oct 19, 2009 2:13 pm
Location: Toronto
Thanked: 539 times
Followed by:164 members
GMAT Score:800

by Testluv » Sun Jun 06, 2010 12:37 am
Rahul,

what the heck are the boldface portions?!

Also, the format in which you've posted the question makes it difficult to read. (Compare the way your question looks to the original question posted in this thread).

Please re-post, and I will happily look into it.
Kaplan Teacher in Toronto

Senior | Next Rank: 100 Posts
Posts: 38
Joined: Wed Jul 22, 2009 8:49 am
Thanked: 4 times

by rahulmehra13 » Sun Jun 06, 2010 12:55 am
Testluv wrote:Rahul,

what the heck are the boldface portions?

Also, the format in which you've posted the question makes it difficult to read. (Compare the way your question looks to the original question posted in this thread).
Sorry for that, here you go :

Although the earliest surviving Greek inscriptions written in an alphabet date from the eighth century B.C., a strong case can be made that the Greeks actually adopted alphabetic writing at least two centuries earlier. Significantly, the text of these earliest surviving Greek inscriptions sometimes runs from right to left and sometimes from left to right. Now, the Greeks learned alphabetic writing from the Phoenicians, and in the process they would surely have adopted whatever convention the Phoenicians were then using with respect to the direction of writing. Originally, Phoenician writing ran in either direction, but by the eighth century B.C. it had been consistently written from right to left for about two centuries. In the argument given, the two portions in boldface play which of the following roles?

A. The first is the position that the argument seeks to establish; the second reports a discovery that has been used to support a position that the argument opposes.
B. The first is the position that the argument seeks to establish; the second presents an assumption on which the argument relies.
C. The first presents evidence that is used in support of the position that the argument seeks to establish; the second presents an assumption on which the argument relies.
D. The first is an objection raised against a position that the argument opposes; the second is the position that the argument seeks to establish.
E. The first is an objection raised against a position that the argument opposes; the second is evidence that has been used to support that position.

GMAT Instructor
Posts: 1302
Joined: Mon Oct 19, 2009 2:13 pm
Location: Toronto
Thanked: 539 times
Followed by:164 members
GMAT Score:800

by Testluv » Sun Jun 06, 2010 1:56 am
The difference is between the word "presents" and "is". If choice B had said that the secondboldface "is" an assumption, then choice B would definitely be wrong. Because choice B says that the second boldface "presents" an assumption, choice B is perfect. (Notice that, here, sometimes the answer choices say "present" and other times "is".)

Here's a definition for assumption: an unstated piece of evidence without which the conclusion will not follow from the stated premises. A statement can't BE an assumption. An assumption is something the author is unaware of. It is something implicit in his argument.

But this doesn't stop the author from discussing or "presenting" to you a consciously made warranted assumption, the force of which he explicitly indicates as support for his conclusion.

Here, the author is pointing out a very well warranted assumption, and because it is so warranted he explicitly invokes it in support of his conclusion. This is not the same as an author who, all unaware, overlooks some premise without which the argument cannot stand. In short: in this argument the author relies on something that he recognizes falls just a tad bit short of being necessarily true (ie, just a bit short of being a valid inference).

We know that the Greeks learned alphabet writing from the Phoenicians. It stands to reason that, at the time of learning, the Greeks "would surely have adopted whatever convention the Phoenicians were then using with respect to the direction of writing". It stands to reason because alternatve conclusions on this point are very unlikely (if the Greeks are learning writing from the Phoenicians in century x, then they are likely to pick up the Phoenicians' xth century conventions, and very unlikely to pick up Phoenician conventions of a different century). However, although alternative conclusions are unlikely, they are not conclusively ruled out. The author recognizes this, so it is not implicit in his argument--it is explicit. While recognizing that it's not necessarily true, he trusts the reader to agree that this assumption is warranted ("would surely have") since it would be bizarre to suggest otherwise.

If you still have doubts, let me know, and I can explain further.
Kaplan Teacher in Toronto

Senior | Next Rank: 100 Posts
Posts: 38
Joined: Wed Jul 22, 2009 8:49 am
Thanked: 4 times

by rahulmehra13 » Sun Jun 06, 2010 2:01 am
Testluv wrote:The difference is between the word "presents" and "is". If choice B had said that the secondboldface "is" an assumption, then choice B would definitely be wrong. Because choice B says that the second boldface "presents" an assumption, choice B is perfect. (Notice that, here, sometimes the answer choices say "present" and other times "is".)

Here's a definition for assumption: an unstated piece of evidence without which the conclusion will not follow from the stated premises. A statement can't BE an assumption. An assumption is something the author is unaware of. It is something implicit in his argument.

But this doesn't stop the author from discussing or "presenting" to you a consciously made warranted assumption, the force of which he explicitly indicates as support for his conclusion.

Here, the author is pointing out a very well warranted assumption, and because it is so warranted he explicitly invokes it in support of his conclusion. This is not the same as an author who, all unaware, overlooks some premise without which the argument cannot stand. In short: in this argument the author relies on something that he recognizes falls just a tad bit short of being necessarily true (ie, just a bit short of being a valid inference).

We know that the Greeks learned alphabet writing from the Phoenicians. It stands to reason that, at the time of learning, the Greeks "would surely have adopted whatever convention the Phoenicians were then using with respect to the direction of writing". It stands to reason because alternatve conclusions on this point are very unlikely (if the Greeks are learning writing from the Phoenicians in century x, then they are likely to pick up the Phoenicians' xth century conventions, and very unlikely to pick up Phoenician conventions of a different century). However, although alternative conclusions are unlikely, they are not conclusively ruled out. The author recognizes this, so it is not implicit in his argument--it is explicit. While recognizing that it's not necessarily true, he trusts the reader to agree that this assumption is warranted ("would surely have") since it would be bizarre to suggest otherwise.

If you still have doubts, let me know, and I can explain further.
It is pretty clear now, thanks for such a great explanation.

Legendary Member
Posts: 2330
Joined: Fri Jan 15, 2010 5:14 am
Thanked: 56 times
Followed by:26 members

by mundasingh123 » Tue Oct 04, 2011 11:31 am
Testluv wrote:The difference is between the word "presents" and "is". If choice B had said that the secondboldface "is" an assumption, then choice B would definitely be wrong. Because choice B says that the second boldface "presents" an assumption, choice B is perfect. (Notice that, here, sometimes the answer choices say "present" and other times "is".)

Here's a definition for assumption: an unstated piece of evidence without which the conclusion will not follow from the stated premises. A statement can't BE an assumption. An assumption is something the author is unaware of. It is something implicit in his argument.

But this doesn't stop the author from discussing or "presenting" to you a consciously made warranted assumption, the force of which he explicitly indicates as support for his conclusion.

Here, the author is pointing out a very well warranted assumption, and because it is so warranted he explicitly invokes it in support of his conclusion. This is not the same as an author who, all unaware, overlooks some premise without which the argument cannot stand. In short: in this argument the author relies on something that he recognizes falls just a tad bit short of being necessarily true (ie, just a bit short of being a valid inference).

We know that the Greeks learned alphabet writing from the Phoenicians. It stands to reason that, at the time of learning, the Greeks "would surely have adopted whatever convention the Phoenicians were then using with respect to the direction of writing". It stands to reason because alternatve conclusions on this point are very unlikely (if the Greeks are learning writing from the Phoenicians in century x, then they are likely to pick up the Phoenicians' xth century conventions, and very unlikely to pick up Phoenician conventions of a different century). However, although alternative conclusions are unlikely, they are not conclusively ruled out. The author recognizes this, so it is not implicit in his argument--it is explicit. While recognizing that it's not necessarily true, he trusts the reader to agree that this assumption is warranted ("would surely have") since it would be bizarre to suggest otherwise.

If you still have doubts, let me know, and I can explain further.
TestLuv ,
I am finding your explanation very complex .
I didnt really understand what you meant by
While recognizing that it's not necessarily true, he trusts the reader to agree that this assumption is warranted ("would surely have") since it would be bizarre to suggest otherwise.
If the second bolded part presents (whatever , i didnt understand the implications of present ) an assumption , then what is the conclusion .
This is an old thread i hope you are still active on BTg
I Seek Explanations Not Answers